USMLE Forums Logo
USMLE Forums
Your Reliable
USMLE Online Community
21,070 Members
72,283 Posts
Falcon Physician Reviews
Home
USMLE Articles
USMLE News
USMLE Polls
USMLE Books
USMLE Mobile
Monthly Contest
Go Back   USMLE Forums > USMLE Step 1 Forum

USMLE Step 1 Forum USMLE Step 1 Discussion Forum: Let's talk about anything related to USMLE Step 1 exam


Reply
 
Thread Tools Search this Thread Display Modes
Old 10-04-2010
USMLE Forums Scout
 
Steps History: Not yet
Posts: 16
Threads: 3
Thanked 5 Times in 5 Posts
Reputation: 15
Heart Forward to Regurgitant Flow Ratio

this is one of the questions of uw...but i cudnt understand the explanation..so plz guys help me wid dis one-

A 46 yr old Caucasian female presents to ur office becoz of easy fatiguability n exertional dyspnea. Auscultation of heart reveals a diminished first heart sound and an apical holosystolic murmur radiating to d axilla. Lungs have bibasilar crackles. There is no elevation of JVP or peripheral edema. Which of d following wud most likely increase forward to regurgitant volume ratio in this patient?

A. Decreasing LV preload
B. Increasing LV contractility
C. Decreasing LV afterload
D. Decreasing heart rate
E. Increasing left ventricular volume
Reply With Quote



Old 10-04-2010
Haisook's Avatar
USMLE Forums Master
 
Steps History: 1+CK+CS
Posts: 683
Threads: 110
Thanked 480 Times in 226 Posts
Reputation: 500
Default

The patient has MR. The question simply asks: what would increase the amount of blood pumped forward through the aortic orifice more than that pumped back into the left atrium.

Answer is C. Decreasing LV afterload.

Because simply when you decrease the afterload (i.e. systemic pressure), there's less resistance to outflow, and so in theory, more blood will likely be pumped forward to the aorta.

A. Decreasing LV preload: decreasing the VR to the LV means a lower left atrial pressure, and so there will be more tendency for regurgitant flow back to the left atrium.

B. Increasing LV contractility: does not necessarily mean that there will be more forward flow, because it would equally lead to increased regurgitant flow, if all other criteria remain unchanged.

D. Decreasing heart rate: would not have a perceivable effect on this ratio. In fact, it may lead to increased VR, increasing LV volume, and more blood going back to the left atrium.

E. Increasing left ventricular volume: see explanation for D.
Reply With Quote
The Following 6 Users Say Thank You to Haisook For This Useful Post:
aktorque (02-18-2011), futureresident (10-04-2010), Mondoshawan (02-17-2011), Rawalian (10-05-2010), Sabio (10-04-2010), wimzie (02-18-2011)
Old 10-04-2010
ashishkabir's Avatar
USMLE Forums Master
 
Steps History: Not yet
Posts: 623
Threads: 19
Thanked 327 Times in 217 Posts
Reputation: 347
Default

Yup, or, otherwise, you can look at it like it is in First Aid...

On p. 255 on the 2010 FA, we find that a "Holosystolic, high pitched "blowing murmur" is either Mitral or Tricuspid regurg. It's not in the tricuspid area, so this is mitral regurg.

Okay - Since we've established that, in the description, it says "Enhanced by maneuvers that increase TPR (squatting, hand grip) or LA return (expiration)

So, since this is a regurg, then we know that anything which increases the murmur must increase the BACKWARD flow and therefore the opposite is true for increasing the FORWARD flow.

So, anything which decreases TPR or LA return would decrease the backward flow and therefore the murmur. Decreasing LV afterload would decrease TPR. Tadaa...
Reply With Quote
The Following 6 Users Say Thank You to ashishkabir For This Useful Post:
aktorque (02-18-2011), futureresident (10-04-2010), Mondoshawan (02-17-2011), Rawalian (10-05-2010), Sabio (10-04-2010), wimzie (02-18-2011)
Old 10-04-2010
dazzles's Avatar
USMLE Forums Addict
 
Steps History: 1 + CS
Posts: 126
Threads: 21
Thanked 95 Times in 33 Posts
Reputation: 105
Default

I'm not that great with cardiology, but I'll try this out.

So you're told the patient has a holosystolic murmur loudest at the apex that radiates to the axilla = Mitral Regurgitation.
By the way the decreased S1 is because your mitral and tricuspid valves close to produce S1 and your mitral valve is not closing that well because of the regurgitation.
You're also told the patient doesn't have a raised JVP or peripheral edema = NOT in heart failure
What are you being asked? You want to increase forward to regurgitant volume = you want the blood to move forwards to the LV instead of backwards to the LA like it normally does in MR.

What happens in MR?
The mitral valve (between the LA and LV) doesn't close properly. If it doesn't close properly then blood is leaking back into the LA during ventricular diastole (relaxation/filling). This means your LVEDV (preload) is lower than normal because blood that would've gone to the LV is able to go back to the LA. This also means you can't generate as high of a pressure as you would want to in the LV during ventricular systole (contraction) because blood is continually leaking back. If your LV systolic pressure is lower than normal then you have to use more effort to get the blood from the LV to the aorta and you're not able to send as much blood to the aorta during ventricular systole. (This is because blood always moves from high pressure to low pressure in the heart so blood can only move from the LV to the aorta when the pressure in the LV is higher than the pressure in the aorta.)
You want to increase the forward movement of blood (LA -> LV -> aorta) right? So either you have to increase your preload (LVEDV) because it's lower than normal in this case or you have to decrease your afterload (Aortic Diastolic Pressure or TPR). You want to decrease your afterload so that the pressure difference between the LV and Aorta becomes higher which allows you to move more blood from the LV to the Aorta.

Someone please check and make sure what I said is correct. Hope this helps you.
Reply With Quote
The Following 4 Users Say Thank You to dazzles For This Useful Post:
aktorque (02-18-2011), Mondoshawan (02-17-2011), steps (02-18-2011), wimzie (02-18-2011)
Old 10-04-2010
USMLE Forums Scout
 
Steps History: Not yet
Posts: 16
Threads: 3
Thanked 5 Times in 5 Posts
Reputation: 15
Thumbs Up thanx

Quote:
Originally Posted by Haisook View Post
The patient has MR. The question simply asks: what would increase the amount of blood pumped forward through the aortic orifice more than that pumped back into the left atrium.

Answer is C. Decreasing LV afterload.

Because simply when you decrease the afterload (i.e. systemic pressure), there's less resistance to outflow, and so in theory, more blood will likely be pumped forward to the aorta.

A. Decreasing LV preload: decreasing the VR to the LV means a lower left atrial pressure, and so there will be more tendency for regurgitant flow back to the left atrium.

B. Increasing LV contractility: does not necessarily mean that there will be more forward flow, because it would equally lead to increased regurgitant flow, if all other criteria remain unchanged.

D. Decreasing heart rate: would not have a perceivable effect on this ratio. In fact, it may lead to increased VR, increasing LV volume, and more blood going back to the left atrium.

E. Increasing left ventricular volume: see explanation for D.
now i get it....i knew d answer just wasnt sure about decreasing d preload thing...but u made it clear....thank u...
Reply With Quote
Old 10-04-2010
USMLE Forums Scout
 
Steps History: Not yet
Posts: 16
Threads: 3
Thanked 5 Times in 5 Posts
Reputation: 15
Default nice explanation

Quote:
Originally Posted by dazzles View Post
I'm not that great with cardiology, but I'll try this out.

You want to increase the forward movement of blood (LA -> LV -> aorta) right? So either you have to increase your preload (LVEDV) because it's lower than normal in this case or you have to decrease your afterload (Aortic Diastolic Pressure or TPR). You want to decrease your afterload so that the pressure difference between the LV and Aorta becomes higher which allows you to move more blood from the LV to the Aorta.

Someone please check and make sure what I said is correct. Hope this helps you.
i get wat u r hinting n it makes sense...so if dere was an option of increasing d preload n decreasing d afterload of LV,both wud be correct???
but how can we increase d preload????
sorry but i m still confused about dis preload concept....
so now my question is if both these choices come....wich 1 shud we choose???
Reply With Quote
Old 10-04-2010
dazzles's Avatar
USMLE Forums Addict
 
Steps History: 1 + CS
Posts: 126
Threads: 21
Thanked 95 Times in 33 Posts
Reputation: 105
Default

I would think that decreasing the afterload is better than increasing the preload because if you increase the preload for a long time you may end up with eccentric hypertrophy (dilatation and hypertrophy) and eventually systolic heart failure. I'm not sure how you would just increase the preload without affecting the afterload or any other parameters.
Reply With Quote
Old 10-04-2010
USMLE Forums Scout
 
Steps History: Not yet
Posts: 16
Threads: 3
Thanked 5 Times in 5 Posts
Reputation: 15
Default

yeah...dats wat i thot too.....
afterload decreasing wud be d better option....thanx
Reply With Quote
Old 02-17-2011
USMLE Forums Scout
 
Steps History: ---
Posts: 82
Threads: 22
Thanked 26 Times in 13 Posts
Reputation: 29
Default

Quote:
Originally Posted by ashishkabir View Post
yup, or, otherwise, you can look at it like it is in first aid...

On p. 255 on the 2010 fa, we find that a "holosystolic, high pitched "blowing murmur" is either mitral or tricuspid regurg. It's not in the tricuspid area, so this is mitral regurg.

Okay - since we've established that, in the description, it says "enhanced by maneuvers that increase tpr (squatting, hand grip) or la return (expiration)

so, since this is a regurg, then we know that anything which increases the murmur must increase the backward flow and therefore the opposite is true for increasing the forward flow.

So, anything which decreases tpr or la return would decrease the backward flow and therefore the murmur. Decreasing lv afterload would decrease tpr. Tadaa...
nice!
thanks
Reply With Quote
Old 02-18-2011
USMLE Forums Newbie
 
Steps History: ---
Posts: 3
Threads: 0
Thanked 2 Times in 2 Posts
Reputation: 12
Default

Quote:
Originally Posted by dazzles View Post
So you're told the patient has a holosystolic murmur loudest at the apex that radiates to the axilla = Mitral Regurgitation.
By the way the decreased S1 is because your mitral and tricuspid valves close to produce S1 and your mitral valve is not closing that well because of the regurgitation.
You're also told the patient doesn't have a raised JVP or peripheral edema = NOT in heart failure
I would just like to make a small comment here. Please note that peripheral edema and a raised JVP are signs of right-sided heart failure. In the patient in question, we are dealing with mitral regurgitation, which (though rarely in isolation) can lead to left-sided heart failure. The mechanism is that regurgitation of blood flow from the left ventricle to the left atrium, will firstly cause left atrial enlargement due to increased left atrial pressure, and then, lead to pulmonary edema. This pulmonary edema can manifest itself on a plain chest film, or be revealed as crackles on auscultation, or, in the extreme case, may lead to paroxysmal nocturnal dyspnea and orthopnea.
In the question mentioned above, the patient is noted to have bibasilar crackles in the lung, which may be an early indication of pulmonary edema and hence, left-sided heart failure.
Reply With Quote
The Following User Says Thank You to abdul.rehman For This Useful Post:
wimzie (02-18-2011)
Old 02-18-2011
USMLE Forums Scout
 
Steps History: Not yet
Posts: 10
Threads: 2
Thanked 4 Times in 3 Posts
Reputation: 14
Default

I have a question to you guys. After reading some of the responses, I was just a little curious, because I dont remember the physio too well.


If you slow down heart rate, are you also increasing preload, because you are giving the ventricles more time to fill?

If you are doing that, you are keeping cardiac output roughly the same as it was at a higher HR. Because CO = SV X HR


Just a little confused.

Also, if you give a selective beta blocker, it is slowing HR, but how is BP decreasing?
Reply With Quote



Reply

Tags
Cardiovascular- , Pathology- , Physiology-



Similar Threads
Thread Thread Starter Forum Replies Last Post
Odds Ratio rasheed USMLE Step 1 Mnemonics 4 04-28-2011 10:28 PM
ERAS interview / application ratio StepTaker Residency Match Forum 8 09-05-2010 12:14 AM
Increased coronary blood flow in aortic stenosis ecgram USMLE Step 1 Forum 2 05-22-2010 02:16 PM
Innocent or Systolic Flow Murmur! Hohepa USMLE Step 2 CK Forum 1 04-05-2010 07:06 PM
Likelihood Ratio kalpana USMLE Step 1 Forum 1 12-26-2009 01:28 AM



Facebook Page USMLE Forums Tweets USMLE Forums Scribd Documents RSS Feed

USMLE® & other trade marks belong to their respective owners, read full disclaimer
USMLE Forums created under Creative Commons 3.0 License. (2009-2011)